Scanned 160201153438 PDF

You might also like

Download as pdf
Download as pdf
You are on page 1of 41
HAPTER THIRTEEN VIRTUAL WORK{METHOD, Vocal Stem for taominng tape bat © Considering beam AD subjected to an arbitrary loading in figure A. deflection at point B is desired which is equal to 4 as shown in figure B, we a virtual unit load at B and it will produce a virtual external work Wee = 1 (8) If we recall from the previous chapter using area moment method, the ¢ slope do of the elastic curve over the differential length dx is expressed as dy | we let m be equal to the virtual internal bending moment acting on the nial length dx, it will produce a virtual internal work as it undergoes the real yn lo as shown in figure D. ‘A= deflection at point B ie apply a unit couple acting at point C, the virtual external work produced by val unit couple as it undergoes the real rotation 8 is equal to Wye = (1) 0 mM wae J fea ° = slope at point © ‘A beam having a span AB equal to 9 m. has an overhang of BC equal to «uniform load of 30 kN/m along the span AB and a concentrated load of! ‘The beam is supported by a hinged at A and a roller at B. Assume El is E = 200000 MPa and I = 400x 10° mm‘ © Compute the deflection at C using virtual work method, @ Compute the deflection at C using area moment method. Compute the deflection at C using ‘conjugate beam method. Solution: © Deflection at C using virtual work method From figure A: EMq=0 9 Ry = 30(9K4.5) + 54(12) Ry =207kN Ry + Ry = 30(9)+ 54 Ry = 3069) + 54-207 R= 1171, From figure C: figure Band C: ou] Limit M 307 p | 0-9 | 117-3 0-3 5x =x mM "rox 3 cats, f Se? ay at oH 9 3 ; aes] + [2] 3 4d, *b Sd, 113(9)3 - 1.25)*] + 183)? = 1275.75 + 486 789.75 Nt 9275 Na 789:75(1000 200000(400 x 10°) 0.00987 9.87 mm. Pollection at C using area moment method Poor figure E 121509) 3 1053(9) Hivn= G20. 3) Wi igrw= 382725 ais & 2194, 791053 Acah-tep Eltee=-"39) (3) El teva =~ 486 bc = 7897511000), -200000(400)10% ‘4c = 0.00987 m c= 9.87 mm ® Deflection at C using conjugate bear method nv a rare seinins riven’ EM, =0 (3 2" 7 4 on, 289 _789() IT EL Ry = 22525, ET 425.25(3)- 24302) ET 789.75 Bh Ee WG detection at C = equivalent ‘moment at C ofthe conjugate bear 7975 No? Ar 71000, §< *sooogos00)108 0.00887 m jc 987 mm ‘Compute the deflection at point D ‘yung virtual work method. ‘Compute the deflection at point D wsing area moment method. Compute the slope at point B wing area moment method. Solution: Peilection at D using virtual work method Place a unit of load at D as shown in figure B. son sural rigidity EI of the beam changes abruj in axial load of 150 kN at the center of the beam. a 1osygyan0794 51 iplly at points B and D. The beam 1504 75x - 1506-6) 7x Joe (soe . Pdemme (ere “a+ dE f Ose a x B x mB xe we]? [25 avo-[@ 5], -12 s]- [eel -[¢ 3], ELap= Bay + FU? - on Rien on (eP- on zor Ely #219375 2193.75 pry nd bp = 28 ka p= 213 Z8q.0008 * 200000(30)108 4p = 0.0366 m Ap =36.6 mm. Deflection at Dusing area moment method From figure D: = #50645), 2251646) $5064), 22545)10) , 225 ‘a= "Jap * EC” 2GE) +” DET vow figure D: a = #50162), 22546 \3)_ AS50011), 225(3)¢7) lone) + 2EL Zee) * EL 5 = 5400, lon 1508 Dy tia a p, 1012560) N° DEL 759878 er Ape i-toya jy 7528.75 -5400 7885. 2198.75 pr md Dy = 1278 kN! ‘jy 2298.7514000)8 * 2ono0orsa0y10® Ay = 0.0366 m, Ay 9 36.6 mm. Hope at Busing area moment method ‘Area of shaded section 2EI +450) 3) 2Q)ET 506,25(1000)8 290000300)10 Ip 0.00844 rad. (slope at B) EG ‘A cantilever beam has a span of 4 m. Itcarries a uniform load of 8 KN/m, © Compute the slope at the unsupported end using virtual work method. ® Compute the deflection at the See so =o a ® Compute the deflection at the Al y ees rs beam method. r ae Solution: Figure C 7 M =~ 4x2 = Virtual bol ; eal Sytem (M a ] Sytem md ; ; oo = 7 From figure B, apply a fictitious unit ‘moment at point A. meet Qe Hl ag) tt 256 ol 256 i (downward) ‘Moment atthe conjugate beam at A Cong eam AINE SAE is equal to the actual deflection at A for the real beain. {cantilever beam having a span of 4m. carries a concentrated load of If free end. Assuming constant EL © Compute the slope at the free end using virtual work. @ Compute the slope at the free end using conjugate beam method, @ Compute the deflection at the free end using virtual work method. Solution: © Slope at B using virtual work From figure A: “The shear atthe conjugate beam is equal tothe slope ofthe real beam. Figwes flection atthe free end using virtual work method ea Sytem) 4 Peis ay Apply a unit couple at Bas shown in: Apply a unit load at B [275] Prater ‘A cantilever beam shown carries a concentrated load of 20 kN at point ‘constant value of E © Compute the deflection at C using virtual work method, ® Compute the deflection at C using. ‘conjugate beam method. @ Compute the slope at C using virtual work method. Solution: © Deflection at C using virtual work method’ _2N Vota System) Apply a unit load at C. Segment | Origin | Limits | _M m cB ic 0-2 | -20% | 109 BA c 2-4 | -20%« | -109 FEZ We have to analyze at section AB since they don't have the same value of El c= Jaa p 20g 240 | SEL EI v (lection at C using conjugate beam method PQ 2 ogo FIO] ET oo we 32 4 Pullection at 20 a. 28 | UGis tre moment at any point in the conjugate beam is equal to the deflection at that point ofthe real beam. lope at C using virtual work method 3s ~— Apply unit couple at C. a { cantilever beam show carries vertical loads of 80 KN and 100 kN at points B respectively and a counterclockwise couple of 400 kN.m. applied at po Assuming constant value of EL © Compute the vertical reaction at C. © Compute the moment at © Determine the smallest moment of inertia 1 required for the beam so smeximum deflection does not exceed the lint of shy af the span length ‘method of virtual work Solution: © Vertical reaction at C Ro =50+100= 150kN @ Moment at C 400 -50(12) - 10046) 1600 kN. ion Real Sytem (40) Yat Sytem) Place a unit load at A. ‘Segment [Origin [Limits M m AB. A 0-6 = 400 - 50x BIO} BC a | 2 | 0-0. 1005- | 109 ann J exte0500 af $00 5-100 +60) 0 6 ' 2 1 fF 400%? | 50x? 150% _ 200x2 Aa [ Il 32 ] a= [2067 + 82] « sc1029 (67) -s091028-671 484 = py {10800 + 75600 - 10800] km? 75600 n= 2 4g = Max. deflection =23; (mn) -75600(1000)3 _ 12 "Some 1 = 11340108 mm sy EN ‘A cantilever beam shown has a span of 9m. It carries @ uniform load of 20 kN/t a clockwise moment of 280 kN.m. at the free end. Assuming EI to be constant E = 200000 MPa. AW ES Jom the given beam shown with diferent moment of inertias and constant value of = 250000 MPa. |) Compute the reaction at A. ‘ompute the deflection at C using the virtual work method. { = 600% 10° mmé. © Compute the vertical reaction at A. ® Compute the moment at A. @ Determine the smallest moment of inertia [required for the beam shown so that its max. deflection should not exceed 25 mm. Use virtual work method. Solution: Solution: Reaction at A © Vertical reaction at A YMy=0 non ON Ray = 20(9) = 180 kN 20 Ry = 100(15) + 200(5) t 4 @ Moment at A By = 125KN sd Vom se? My =~ 280 - 200945) =~ 1050 kN. y= 200-100-125 fs “Ft Ra 175kN ® Moment of inertia vs) Ryl75 = a0 206069 Sytem) Detection at € uN 2010 YMa=0 : — [et = fl an JP 0289-100 5 2 Robs m075 pe ml pera) 1 280x? | 10x! id Place a unit load at C. EI 2 t 4 lo ; 4 ii 09 +2] L = ™ = Sem) “apply aie oad at as | a | os 175. 05x 1s [a7sx-2006-5) | 025 1025 m, = 27742-51000) a caer — DC _D 0-5 125% 0.75x. 1 5548.5 x 108 mm 532 4 -f (o2seyi7ieddx , Jvo2ss 175% -200)(x-5)ax , f c= " ’ oe see 6250 , 2502] , [93.758 7° tex “er J, * Laer J, J (5) ers J (35) 075-205 ba ET * Er fe = 182292 | al 12500 , 3906.25 ope lana Bly EL BAT? c= 1944.75(1000 = Fanon Ac = 0.09897 mm 1 “Pel El sas El Er 260.42 and Ac = 9897 m. Slope at C using virtual work method 1927,08(10009 *250000(600)10° (= 0.0128 rad, Apply a unit couple at C. IM, =0 20Rp=1 A beam shown carries a uniform load of 2 KN/m throughout its length. The beam is pin coruected at A and a roller at B. Ascume Bl ic constant to be 6200 kN? ‘We have to consider section BC since they don't have equal values of EL 2k Se ) Compute the reaction at A. ) Compute the deflection at C using virtual work method. } Compute the slope at C using virtual work method. Segment | Origin [Limits M AB A 0-5 15x 5-15 | 175x-200x-5) A 125 D Solution: © Reaction at A foes ! 2Mp=0 sCCCLLECE CE, 15 Ry = 2(21)(4.5) : - By =2264N Slope at C Place a unit couple at C. © Deflection at C Se Apply a unit load at C EMg=0 | 0.067%. a Ry (15) = 106) Ry =040kN Wert Sytem m) Place a unit Ioad at C. ose eae] Le, 94973 , 1797 (c= - 0.0087 rad. | beam having a span of 12 m. carries a concentrated load of 60 KN at point B Jhssuming Els constant to be 50000 kN? and using virtual work method, own 1) Compute the deflection at C. Compute the deflection at B. ) Compute the slope at B. ea c=-0.045m. T upward Ac = -45 mm, 2 Deflection at B 2Mp= 12Rp=10) Ry=075KN Re O.75 en RABIN Place a unit load at B. 45x - 60(y.-3) = 180 15x =a ReiS kN Maru Sem (m) Place a unit load at C. se Pasgene,f esmog ss, espn tf [3], Ee] Ler], Lal 225+ 105-4725 —ae ne 3 1) Compute the moment at A. ® Compute the horizontal reaction at A. ) Compute the vertical deflection at joint C of the frame using virtual ‘Segment | Orig Limit M m work method. AB A 0-3 ax | -< 2 Solution: DB D 0-9 we | + J D ) Moment at A My =25(4) + 26608) a My=136KN.m. es= J "Er ax ae ox sf (& jesow (4) asoex 7 i ° o ~- Lae] [ 1258 7° 3E lo ® Horizontal reaction at A. Ran =2(6)=12KN Base Workcon Frames | The frame shown is subjected toa lateral uniform load of 2 kN/m along member AB Anda concentrated load of 25 KN at C. Assuming EI to be constant with Place unt couple at B. 200000 MPa and = 800% 108 mm ase ail ln om 1OOKN. aie to we Vm 2 sim C 2m] Lon 2eNInf 13k ea Sytem iM) agen 136K @ Vertical deflection at joint C Apply a vertical unit load at C Segment BA ce ‘he frame shown is subjected to a lateral load of 20 kN at joint B and a uniform load (3 KN /m on member BC. Assume EI is constant with values of E = 200000 MPa and + 400x 108 mm, Using virtual work method, 3m 1) Compute the horizontal reaction at A. Compute the moment at B. 4) Compute the horizontal deflection at joint C of the frame. Solution: ') Horizontal reaction at A Ran =20KN Moment at B YMa=0 10 Rp = 20010) + 3(10}5) Ry = 35 kN Ry = 3010) - 35 Ra My = 200 kN. @ Horizontal deflection at C Apply a horizontal unit load at C. rey [ae 1458333 5883.33 \Nm? 1000 "20 9000(400)10° Ac =0.182m. Ihe fame shown has a pin support at A and a roller at D. It caries a uniform load of JAN /m along member BC. Assume constant value of El = 200000 MPa 1» 1365108 met. Using virtual work method, ) Compute the horizontal reaction at A. 9 Compute the vertical reaction at D. 4) Compute the horizontal deflection at joint C. Solution: 1) Horizontal reaction at A. Raw = 1) Vertical reaction at D KN (vertical reaction at D) Ry =3(6)-9 Ry = 9kN Horizontal deflection at C Place a horizontal unit load at C. IMa=0 Rp(6)=102) Ro= _224(10009_ 4c =Z00900c136)108 KN 4c #001191 m. c= 1191 m, Wea © A fame showin fas trl ond of 20 KN ating at jon. Assuming El with E = 200000 MPa, a ns © Compute the vertical reaction at A. '® Compute the total reaction at A © Compute the smallest moment of inertia required for the members ofthe f that the horizontal deflection at joint C does not exceed 25 mm using virtual method. Solution: ‘Vertical reaction at A YMq=0 196) =F) Ry =30KN iy =30kN Total reaction at A. Ra = 201N eal yen 80 5 Ry = V0? + G0? Ry =42.43 kN Horizontal deflection at C Apply a horizontal unit load at C. ment] Origin [Limit [Mt awit a | 0-6 | am cB c | o-6 | 3% ‘mM so= PY sea ff coms, fF wore JET > 308 wey. tc= [3 + OE SE 7 HE I) inten Ac a mr uns = zagoone 1 = 864 x 106 mm From the given frame shown: Which of the following gives the reaction at D. Which of the following gives the reaction at A. Which of the following gives the rotation at joint D. Solution : Reaction at D 2Ma=0 Rp (4) = 4502) Rp=90kN Reaction at A rT aimyeoef) po8( fx) a0 E1@p= J 52s 154 ° El0p= ‘ ‘ 52 | ssa | 0 0 EL @p= - 480 + 360 120 00003 a 8 F500 25x 105) “OBS Op= -0172" cca The rigid frame shown is supported by a hinge at A and a roller at D and carries the Iwiangularly distributed load. Assuming constant EI = 52000 N-m? and neglecting vial deformation. ® A Which of the following gives the Which of the following gives the reaction at roller D. Which of the following gives the reaction at support A. horizontal deflection at D. Solution: Keaction at D DMa=0 Rp 3)=361900)3(3) Brve0 [221] Protien Ri=0 A rigid frame, loaded as shown is hinged at A and supported on rollers at D. Ry = V0? (507 Assuming El is constant throughout the frame and neglecting, the effect of axial {eformations. eons wan Pacman date lr Jim. iets al Which of the following gives the noe I. ra each pt I horizontal deflection at the roller D. | Solution : 4) Reaction at Ds Ma =0 2Rp = 8002K0) sdonm Rp= 800N ® Reaction at A: 3Ry=0 Ra +Rp= 8002) | Horizontal deflection at D: Place a unit load at D. TMaso 2Ry=10) snes [Stor Sot. Wor HMeae] sone 4052000 4p =00571m. Ap= 57 mm Rein 169) p= 5) Mad ap=0+gr f (600% - 400%2) (4-3) ° 4p=0+ar F ote 10002-102s2002 oe ° ane fy [202 0 4002 , amet]? wre 2 3 3 4 [22] Pot From the rigid frame shown, if El = 10x 10!2 N.mm?. © Which of the following gives the reaction at E. @ Which of the following gives the reaction at A. ® Which of the following gives the horizontal displacement at D. Solution : 1) Reaction at E: =Ma, SRE = 60(3) Rg=36KN D Reaction at A: Rat Re=60 Ra = 60-36 Ras 26kN 1) Horizontal displacement at D: Place a horizontal unit load at D. alee] abe] Which of the following gives the reaction at D. Which of the following gives the reaction at A. Which of the following gives the rotation of joint C in radians. Solution: Reaction at D: BMa=0 Ry = 10(3)1.5) + 2002) Ry = 28.33 Reaction at A: BFy=0 Ry + Rp = 103) Rp = 30-2833 Ro =167 3Fh=0 R3=20 Ra = Verner? Ra =2007 kN 2) Rotation at joint Cin radians: Using Virtual Work Method Myo=E J BMS Apply a unit moment at C: SMa =0 R@)=1 oe fe 2, en | ST ® -d) [222 7 SET 3 4 dy vee [232-594] ghia t 512. i 51.24 3970 0¢ = 0.0129 radians Bys0 =RE=15 ‘The frame shown is pin-connected at A and supported by a roller at C. It is: parc to a horizontal uniformly distributed load of 18 kN/m along AB. 2 =a E = 200000 MPa, 1=250% 106 mim# ie mene AB EMOse fi © Which of the following gives the = 108e- 18000 Rents reaction at C om ®@ Which of the following gives the 18 M = 108x - 9x2 reaction at A. kyo om m=10) I © Which of the following gives the r horizontal displacement at C. Solution: For member BC: from 0to4 © Reaction atC Beast Baris EMA=0 m=15« 4Rc = 18(6)9) Ro=stkn © Reaction at A: IF_=0 Ray = 1866) J cme. ii (1xy(LS50)dx Rax = 108 kN. i ° rn 6 choo wb PET a Pe], Ray =Rc=81 KN co [xet.204] +f [2] Ree aasame a Ras 135eN hen iP wat 745210007 _ "200000(250) 108 ® Horizontal displacement of C. PPlace a horizontal unit load at C. 3Ma=0 ARc = 1(6) Rea ET From the given frame shown itis subjected to a horizontal oad of 600 N at join a vertical concentrated load of 900 N at the midspan of BC. Assuming equal El forall members. 900 N © Which of the following gives the reaction at D. ® Which of the following gives the reaction at A. Which of the following gives the horizontal displacement of joint D. Solution: Reaction at D: Ma =0 Ry (4) = 9002) + 600) Roy = 1200 Ra = ¥(600)? + (300)? Ra =67082N Horizontal displacement at D: For AB: M=600x m=10) For BE: IM = 600(5) - 300% M-= 3000 - 300% = 166) 0.5060) m=5-05x For CE: M= 1200% m= 1G) + 0.50 For cD: Meo moi) a 10 Reso 1ap,=2 J Mae Preeti. ° EMco 4050 Vang Feats [ 0-20098-055)4 ° ’ + f (12003)(3+0.505) dx + F cova ° a mon. Joan2an+ Faso?-smoes sam a+ Fasano a+0 ° ° o cand], [25g smn J. [peep 1 o0= [S92] 92-2 som] [BPE ET" 0 2 ssoes? manos? oe Wy, = 2982 OOH, 20000755090)» MOB aon Wh dry, = $8200 Nim Hor, = 58.2 Nn? Ena [A frame shown is pin connected at A and a roller support at E. It ‘concentrated load of 30 kN at C, a uniform load of 3 kN/m along member DE lateral load of 40 KN at mid height of aN ‘member AD, Assume E= 200000 MPa, 34m 1= 4000 x 10 met, © Compute the vertical reaction at A. @ Compute the rotation of joint D of the frame using virtual work method. © Compute the horizontal deflection of joint E using virtual work method. Solution: © Vertical reaction at A EM, Ry @) + 3068) = 40(4) + 3X8) Ry=17KN 8)+30-17 y= 37kN © Rotation ofjoint D Ports w=) a y= [06258 ours i ad, 8533 pre = SH aN 85,33(1000)° 2000004000) 0.000107 rad. Deflection of joint E Apply @ horizontal unit force at E. YMp=0 Re @)= 18) Real a7 eal Siem (8 ira Sytem (m) M 0x Tw frame shown has a fixed support at A and is subjected to a Iateral force of 20 kN v4 30 KN at joints B and D respectively. IE = 70000 MPa and I= 1248 x 10° men. 40x 40(x 4) = 160, 1 Compute the horizontal reaction at A. 204 ® Compute the moment at A. ¥ Compute the rotation at joint D using virtual work method. 17x-8 21-158 a Solution: 1) Horizontal reaction at A oe [nt q 8 8 > = ace J’ gon, J? xasmax, |? x07%-150) ax Ran 20+ J) et det td, Ue see [°]- [8] + Pe] Le ser J, * Ler J, * [Ser J [set 42667 , 1900 , 5067 768 EL EL BL” ET ® Moment at A Ma #3008) +2016) Ma =210 kNam. © Rotation at joint D Apply a unit couple at D. im Apply a unit couple at D. [Segment | Origin | Limit M m AB A 0-6 | -210+50x 4 BC. B o-4 0 a ve | pb 0-3 = 30% 1 mM. op= JH ax rs i 7 — fF crrer10 sony f a eo, f ( — i; 1osoode', | cre ue soe 0 6 a ee oo [ 39" | [3] [F] [2] at J," ae J, jo Ler J oy = 620.450. 360 135 = EI EL EL aes 0p =~ Bp ka Op =_231St000 Fon00ct 24810 Op = - 0.0036 rad. is subjected to a lateral load of ving a fixed support at A and ye shown is having a fixed support al en of int Band an inclined uniform load of 12 kN/m. along the incline 4 "1030 x 10° mm? and using virtual work method, 1» 70000 MPa and. yypute the horizontal reaction at A. gate the moment at A. sypute the vertical deflection at joint C. Et = constant S 126) 40 ; | wa(3) +40 | veh a ntat A 6) (3) 48kN sow 40(5) + 482) + 36165) Rye T68 © yt “ eal sytem 80 5 @ gelato Vinal Sytem) © Deflection at C Apply a vertical unit load of C. [Segment [Origin [Limit M a AB a_| os 76520 4 ce : ae £ or F=-62 | -Zax Joye Hr & ee 3082]° ne ey] Lf ac =- S30 go c= 72 ene? c= Ac =01047m, c= 1047 mm. fPesomzsom fC 0 a 0 Tel 120% El 750, ET 2axt 5 @@et], fame shown is pin connected at A and supported by a roller at C. It carries a ical load at the mid point at AB and a uniform vertical load of 20 KN/m at jit BC, Assume constant value of EI for all the members with E = 200000 MPa 1 - 500% 108 mm. Using virtual work method. ‘Compute the vertical reaction at C. Compute the rotation of joint B of he frame, ‘Compute the vertical deflection at Joint B ofthe frame. Solution: Vertical reaction at C YM,=0 he = 150015) +205) 5.5) 96.875 KN Rotation of int B Real Space 8 I deflection at B Apply a unit couple at joint B vertical unit load at B. B= Re (8)=1 Re =0.125 kN za Oi [Ti a> [a [0-25 sm | 0 a ra co every ci 23 ae b,-0 ce [ic | os || searsn 22 biaip) os =u oJ See 5 {ge PPewomgumae, f* cain gs 58 A ni ; bas : ar 2s « Porsasegs. ca | bee Peers f ° fie esrmguome, | asaagr2n f ft 0575)96 875-10 [uel wT: 1375 y= 1@o140008 SET ser d,,* Lo 2et 200000500108 a7 0) = 0.0071 rad, 79.43 + 25.63 + 791.02 + 1513.6; ET 192381 py 3 BE sm a= 192381010009 "= 200000(600)106 43=0.0192m. p=19.2 mm. From the given frame shown itis supported by a hinged at A and a roll carries a horizontal uniform load of 4 KN/m acting on member AB. Us ‘work method of analysis. © Compute the vertical reaction at C. @ Compute the moment Mga. ® Compute the horizontal displacement of C assuming E = 200000 MPa, I = 2500 x 10° mint Solution: © Vertical reaction at C 3M,=0 BRey = 4UY}) Roy =25 kN. ® Moment Mga Ray =4(10) =: Maa 40kN 40(10) + 4(10)5) 200 KN, oO 25kN eal Sytem 0) Pisce a horizontal unit of load at C. YM,=0 ky =100) Hy © 1.25kN Wy = 125KN Limit enn Origin Pee | a0 1) 125% c_| o-8 mM er ik (dn 22d f .z5qpiade fel! Eel a] dou= dou q “amseri oO dey = 0.0278 fey) = 273 mm. 569 a P 4 B c ea Sym (1) y= (iy, ‘$= intemal normal force ina truss member caused by real loads U-= internal virtual normal force in a truss member caused by external vi load placed at the joint whose deflection is required Length of members ‘A= cross sectional area of a member F = modulus of lasticty of a member |A, = vertical displacement or deflection caused by real loads onthe rss ‘4, = horizontal displacement or deflection caused by real loads on the t WE Fi the truss shown in the figure, itis subjected to a horizontal force of 10 kN at WB. Assume AE is constant with E = 30000 MPa, A = 500 mm? using virtual work tod, Compute the vertical reaction at C. Compute the horizontal component of the deflection at joint B Compute the vertical component of the leflection at joint B. Solution: Vertical reaction at C YMa=0 O Rey = 10¢4) Ney = 667 kN Horizontal component of deflection at B tr,-0 ie (£) «607 We-8311N Gomp) nts D0 )y(8)-9 AV 834KN (tension) 2 10KN 4 Rey=667 EN Place a horizontal unit load at B, Vertical component of deflection at B G EM, =0 Place a vertical load unit at B. 7 enn ; 2 «066718 we($)-08 fA ne=06254N (eompresson) / 7 v¢(4) «607 AB=O5254N_ (ompresion) ost \ 5C=084. (amp) AS A =e an ($) «6607 ain stn ‘AB=0.834 (tension) a =26.05, + 26.06 ESUL=0 | se sau | sos | 3470 ee Pe ee SUL = 69.56 kN?.m. ‘0 (vertical component of deflection at B) EM,=0 5Ri=1@) Ry =06KN (Comp) Ry=06KN (Tension) From the given tras shown in the figure itis subjected to a horizontal load of at Band a vertical load of 40 KN at B. Assume EA as constant with E = 20000 and A = 1000 mm2, Use virtual work method. 40KN © Compute the vertical reaction at C. @ Compute the horizontal component of the ay 201 deflection at B. ABQ) og @ Compute the vertical component of the 424 deflection at B. AB = 0.848 (Tension) Solution: 08180) Oval tennuc =e ace 8 cles AC=040KN (tension) 5 Rey=209) +40) Roy =36kN BF, =0 ele) zer = 060 BC=0.722KN (comp) omen @ Horizontal component ofthe deflection at B Ray + Rey =40 Cheek: 0.720) 720) 040 (OK) AB=SESKN (amp) x5 _| am | 56 | +088 | 2031 xc_| sam | 0» | om | +ima m0 wie A ca | stom [2 | +0 | +0 w (sr) = ae ye YSUL= 14060 kN? im. BC=4332 (comp) SUL. (1) 4a, = 2B ZA=0 rr : oan nc=ses (37) +20 ees AC =24 (tension) 3g, =0.7 mm, —> (to the right) Place a vertical unit load at B. EMa=0 5R3=10) AB=0565 KN (comp.) Fh=0 Ac AC=040KN (tension) Try=0 BC) _ 361 7060 ® Vertical component ofthe deflection at B 576 (ag, = ESE 174.45(1000) (88, = 260000(1000) 5, = 00087, 4,-08rmm (doweward) Cie) rom the given frame point Cis supported by a roller while A ifhinged. i A mm: Benin 1) Which of the following gives the stress of BC due to actual loads only. ® Which of the following gives the stress of BC due to a vertical unit load only. © Which of the following gives the vertical deflection at B using virtual work method. AB=1x106 KN BC=0722%N (comp) Solution: Sires of member BC. AC gc AB_B_ 6 Checks Maso ac =0220) 4009) + 60000) =4R 361 Wy = 200K AC= 0808 (OK) srh=0 = CSin@ = 200 mba [1 5 u a ssuL_] v8) (600) Sam ee tall | aes | geri) ee | C= SOK () sc {sa | oe» | om | win (oe ie ee ele SUL = 174.45 kN? m, 578 (ea wenTTAND) —_—————$—$—$— Paz a Thvee structural member (AB, BC and CA) of equal lengths, L, areas A and elastic nocluli E, pin jointed at the ends to form a triangular truss ABC. The truss is pinned (not roller) supported at A and B. At Ca vertical load w is applied at C. Which of the Jollowing gives the vertical deflection at joint C. Solution: [AB does not deform since they are pin 1G) =R2 (4) Rp =0.75 AC RC connected. Only AC and BC will deform. 6 AC = BC BC=Sind = \ see-a7 wh voewinew reaeaege acai (comp) BC = 1.25 (-) 3 AC=125 Cos0 nas ac=1 a) A AC #10 (9) AB = 1.25 Sind 1251.0 AB =150)_ 075 (+) Stress of BC = 1.25 © Vertical deflection at B: ag = 55 (189 = {Zp = 000675 m. OB = 6.75 mm. $79 580 EAM aa eas Solution: 1) Horizontal reaction at A ‘The pin-jointed frame work shown on the figure below has members of YMc=0 sectional area A = 1200 ram? and Youngs Modulus of 20500 N/tnen2, Which 502) Sin 60" = Rags 2) following gives the vertical deflection ofthe load. gt = 830A Solution ® Horizontal deflection at B pumocet Place a horizontal unit load at B. Mc =0 (0)@) Cos 60"= R, 2) an : ‘An equilateral triangular truss ABC has a vertical member AC that has one of it : 2m. long, Ithas a roller support at A and hinged at C. The truss carries a vertical pemeecirmaal of 50 KN af joint B. Cross-sectional area of all members te 800 emt 1289 (comp) = 200,000 MPa. ‘ABSin 60°=43.30 © Compute the horizontal reaction at A. AB=50KN (tension) @ Compute the horizontal deflection at B. | ® Compute the vertical deflection a B 50Cos 60" =25KN (comp) a ac BC'Sin 60" = Cos 30" BC=50KN (comp.) ee sow 582 a= 2 4057} a7 (1) (oa = sppesaaeeoy =0269 | +1445 (ESUL = 14.45 kN2.m. (abv = 0.0041 m. (aby = 1.41 mom. ©) do BSE H 1. ! 1) ahi» sate} 307] Problem (48x = 0.0000903 m ‘The frame shown is supported at A and B by hinges and is subjected toa horizontal Oakes Oa load of 100 KN at joint C and a vertical load of 50 KN at joint C and joint D spectively. Assume constant value of EA with E = 70000 MPa. Using virtual work ® Vertical deflection at B preston. Place vertical unit load at B. 1) Compute the vertical reaction at A. ® Compute the horizontal reaction at B. Compute the smallest cross sectional =0 P Me area "A" required for the member of the i 2R1=12)Sin6o" truss so that the horizontal deflection at i Ry = 0866 joint D does not exceed 10 mn. ABSin 60" = 0.866 solation: AB=1.0 (tension) 1) Vertical reaction at A / AC + 1.0.Cos co" EM, =0 AC=050 (comp,) 3 Rpy = 100(4) + 50(3) BC=1.0 (comp) Rpy\= 183.33 kN ac Mente Hay [ Su R= 50+ 50+ Ray } ap 2 [ss [10 Ts 100 Ray = 1833-100 pe [2 =50 | -10 [100 Ryy= 8333 kN ca [2 =25 | 050 [425 RyABBIN Ry lB8334N SUL = 225 kN? m, Horizontal ection t 8 3c (4) =183.39-50 BC = 166.66 kN (comp.) BFy\=0 180) Ry Bag 100 AN © Cross sectional area for the member ofthe truss Place a horizontal unit load at D: EM, =0 BR, =1(4) Ry =133kN (§)-19 BC=1.66KN (comp.) y=166 (3) Ry =LOKN Ry = ON UL, ac | 4m | +8333 $443.32 co | 3m o +10 °. pp_| 4m | -s0 0 © 16666 | 1.66 + 1383.28 SUL = 1826.60 kN?.m, a ‘The frame shown is subjected to a lateral load of 100 KN at joint C and a vertical oad. 01 50 KN each acting at joint C and D respectively. The frame is pin connected at A sh supported by a roller at B. Using virtual work method. Compute the vertical reaction at B. son sou ® Compute the horizontal reaction at A. ) Compute the smallest cross sectional cd area "A" required for the members of 4 the truss shown so that the horizontal deflection at joint D does not exceed 10 mm. Assume EA is constant with jan = 70000 MPa. Solution: sou sou 3 Ray = 50(3) + 100(4) | Ryy = 288.33 kN } Horizontal reaction at A wra=0 Ry =200 RN RyABBSBEN Ryy=l83394N © Cross sectional area "A" BF=0 ADSin 0 +50 = 18333 ©, Ib (1yc01 en = 2358680000) AD = 166.67 kN (tension) a 183338N (ym) = R= kN. Bh=0 a 53000 CD~=16667Cos9 ‘wie (noo = ott co=16657(2) 3370? CD=100kN (comp.) som fay #33kN 33) AC=50KN (compression) Oa = coe ” Which of the following gives the ae ernie am lad deflection at C when the cross- sectional area of the member is 1500 mm2. E = 200 000 MPa. Ie there isa support at C, which of the following gives the equivalent reaction so that there will be no Aefletion at C My =0 Ry, @)=104) Ryy=133KN (comp) Ray 133 KN (tension) ADSin@ = 133 Solution: Detection at C: 40(§) 1 Stes deo actual loads: YMa=0 AD = 166 Ry GB) = 9006) ity = 180 kN AteintD: DC=180kN (Comp) Atjoine Bs BCSin 45° = 90 BO=127.28 KN (Comp,) BE = BC-Cos 45" BE =90 KN (tension) 588 587 ETE : Atjoint E: A, ‘ soe aia oat ome cas AE=EB £901 (tension smo “ote ACSin 45 =90 ap] 27.28 KN (tension) x th Neth A oP 7 me Due to the unit load: me Rir= R=! Placea unit load at C ff RECs chee R 4 AUR 1 le uaa R=1 lo Rat Mc o z ADs i AD=0 9 cD=0 Ri) Ly ug Ry =1=RQ=R3 ACSin 45 =1 ac a a AC=1414 tension) Atjoint AE=-ACCos45+1=0 GPE ell [sm FH _ et) BC Cos 45+ 1 = AC Cos 45 a. ; DE=Ri = Gen BC Cos 45 +1 = 1414 Cos 45, iw t— BFy=0; AD= eae Atjoint C: EFy=0 CE +1414 Cos 45 21 2 nN ) ACSin 45 CE=0 ) KC =1414 (comp.) q Tabulation: Members {Length | Area] A u Members | Length: |" Area A u A aE, 3000 | “1500 [5000 To 0 2 0 0 3000_ 1500 EB 3000_ {1500 | +90.00-| 0 0 a 0 - Bc J soov3 {1500 | 12728 | 0 0 wi] s000v3 | 1500 | 0 o. ~ cof 3000 {1500 [= 180.00 | 1} 60 aD a | ‘AD {3000 —|“iso0[— 0 0 0 ota 0 0 Ac] so00v2 {1500 | si2728 | siais | Soe eT soon Tata _| S55, 0 EC 3000 [1500 [0 —L A o— 4¢= 2 SUE a = ESE 3 SUL_ 7655 869.04 x 103 deflection at = 869.04 10% 3452 mm say 4.34 mm Upward deflection y deflection at . 589 To obtain zero deflection at C, downward deflection » upward deflection 7.655R FOR a3 R=113390N R=134kN A steel truss shown has a cross sectional area of each member of 300 ma? ‘modulus of elasticity of 200000 MPa. © Which of the following gives the bar force of member BE due to actual ay 3 Toads. y ® Which of the following gives the bar force of member BE due to a vertical unit load at C, au ® Which of the following gives the vertical displacement of joint C, Solution: © Bar force on BE due to actual load Bar force on BE = 0 due to actual load. tng HB ® Bar force on BE due to unit load Bar force BE = - 0.471 kN 590 Vertical displacement at C jember a ES PEED +s | +0333 TT z982 acme |e +18 [+0607 [ass cD 3 +18 +0667 | 36018 DE 424 = 25.46 E 0471 BE we [3 +13_| +033 _|_ 17962 ar_[ aaa | -546 | our | sous io 3 +18 z St) {SUL = 992.624 EN hoo BE pen B22 1 © *"300(200000) 5.54 mm rom the figure shown: ) Which of the following gives the bar force of member CD due to the actual loads. Which of the following gives the bar force of member BD due to vertical unit load at A. ) Which of the following gives the vertical deflection at A OG sey it Solution: © Bar forces due to actual loads by joint method pie Paton ‘etre shoven has its member CF equal to 15 mm too long and member EF 10 mm se crm. Using the method of virtual work, compute the vertical deflection at (CD =2828 kN pin D. @ Bar forces due to unit load A by joint method BD=1.0kN @ Vertical deflection at A: U (KN) wu 15kNnm. 41 =10 kN. YUS=-25kNanm. SUL Vert an = 5 SU (yA=-25 519(108) Vert by = 2000) 25mm T (upward) 30x10 Vert A = 17:30 mm EW) i Determine the vertical deflection at joint of the truss shown due to a temp drop of 10° Cin member EF and a temperature increase of 30° C in memba (Coefficient of thermal expansion a= 11.7 x 10°8/C” Solution: 8) Deflection of Beams er aM) yy ax i ff Qi a P = applied vertical or horzonial load atthe joint whose deflection is em ae sia Real Sytem (7) }) Rotations of Reams Member [ Umm) [ ar) | UG) Ua I ae E EF. =] 10" 1 | o- | (¥) ue 3 +1 | 60000 : cr ‘6000 +30" 1 180000 EU jb = 2400 mam = applied fictitious moment atthe joint whose rotation is required. (A=Zauat) (1) 4=11.7x 10-6 (240000) 4=-281 mm T (opreardy EE tex: 1a) Deflection of Frames: (when axial deformation is considered) 26D 8 {GD e

You might also like